Skip to content
Snippets Groups Projects
Commit 313ee0c8 authored by José Antonio López Rodríguez's avatar José Antonio López Rodríguez
Browse files

primeros archivos

parent 7306d53b
Branches master
No related tags found
No related merge requests found
File added
%tomado y transformado de: https://www.overleaf.com/latex/templates/assignment-homework-solutions-template/nfjybpqbdrrn
%%%%%%
%% No edites esta parte
%%%%%%
\documentclass[11pt]{article}
\usepackage[papersize={10in,6.5in},margin=0.5in]{geometry}
\usepackage[utf8]{inputenc}
\usepackage[spanish]{babel}
%%%%%%
%% Opciones
%%%%%%
\usepackage[fleqn]{amsmath}
\usepackage{amssymb,amsthm} % assumes amsmath package installed
\usepackage{xcolor}
\usepackage{mdframed}
\newenvironment{problem}[2][Problem]
{ \begin{mdframed}[backgroundcolor=gray!20] \textbf{#1 #2} \\}
{ \end{mdframed}}
% Define solution environment
\newenvironment{solution}
{\textit{Solution:}}
{}
\renewcommand{\qed}{\quad\qedsymbol}
%%%%%%%%%%%%%%%%%%%%%%%%%%%%%%%%%%%%%%%%%%%%%%%%%%%%%%%%%%%%%%%%%%%%%%%%%%%%%%%%%%%%%%%%%%%%%%%%%%%%%%%%%%%%%%%%%%%%%%%%%%%%%%%%%%%%%%%%
\begin{document}
%%%%%%%%%%%%%%
%% Datos de la tarea: mantener formato
%%%%%%%%%%%%%%
\noindent
%%%%%%%%%%%%%%%%%%%%%%%%%%%%%%%%%%%%%%%%%%%%%%%%%%%%%%%%%%%%%%%%%%%%%%%%%%%%%%%%%%%%%%%%%%%%%%%%%%%%%%%%%%%%%%%%%%%%%%%%%%%%%%%%%%%%%%%%
\large\textbf{Nombre} \hfill \textbf{Tarea - \#} \\
Mattermost @nombre \hfill ID: 123456789 \\
\normalsize Curso: M* - TemaTema \& Teoría de Campos \hfill LA-CoNGA 2021\\
Responsable: XX. XXXXX \hfill Fecha: \today \\
\noindent\rule{\textwidth}{2.8pt}
%%%%%%%%%%%%%%%%%%%%%%%%%%%%%%%%%%%%%%%%%%%%%%%%%%%%%%%%%%%%%%%%%%%%%%%%%%%%%%%%%%%%%%%%%%%%%%%%%%%%%%%%%%%%%%%%%%%%%%%%%%%%%%%%%%%%%%%%
% Problema 1
%%%%%%%%%%%%%%%%%%%%%%%%%%%%%%%%%%%%%%%%%%%%%%%%%%%%%%%%%%%%%%%%%%%%%%%%%%%%%%%%%%%%%%%%%%%%%%%%%%%%%%%%%%%%%%%%%%%%%%%%%%%%%%%%%%%%%%%%
\begin{problem}{1}
Considere la acción del Campo de Klein-Gordon
\begin{equation}\label{accion.campo.escalar.real}
\mathcal{S}=\int {\rm d}^4 x \mathcal{L}=\frac{1}{2}\int {\rm d}^4 x \left(\partial_{\mu}\phi\partial^{\mu}\phi - m^2\phi^2\right).
\end{equation}
Halle las ecuaciones de movimiento ...
\end{problem}
\begin{solution}
Las ecuaciones de movimiento se derivan del principio variacional, bla, bla ... acción definida en \ref{accion.campo.escalar.real} ... definimos $\Box \equiv \partial_{\mu}\partial^{\mu}$ ... expresión \ref{ecuacion.KG}:
\begin{equation}\label{ecuacion.KG}
\left(\Box + m^2\right)\phi=0.
\end{equation}
\end{solution}
\noindent\rule{\textwidth}{2.8pt}
%%%%%%%%%%%%%%%%%%%%%%%%%%%%%%%%%%%%%%%%%%%%%%%%%%%%%%%%%%%%%%%%%%%%%%%%%
% Problem 2
%%%%%%%%%%%%%%%%%%%%%%%%%%%%%%%%%%%%%%%%%%%%%%%%%%%%%%%%%%%%%%%%%%%%%%%%%%%%%%%%%%%%%%%%%%%%%%%%%%%%%%%%%%%%%%%%%%%%%%%%%%%%%%%%%%%%%%%%
\begin{problem}{1}
Considere la acción del Campo de Klein-Gordon
\begin{equation}\label{accion.campo.escalar.complejo}
\mathcal{S}=\int {\rm d}^4 x \mathcal{L}=\int {\rm d}^4 x \left(\partial_{\mu}\phi\partial^{\mu}\phi^{*} - m^2\phi\phi^{*}\right).
\end{equation}
Halle las ecuaciones de movimiento ...
\end{problem}
\begin{solution}
Las ecuaciones de movimiento se derivan del principio variacional, bla, bla ... acción definida en \ref{accion.campo.escalar.complejo} ... definimos $\Box \equiv \partial_{\mu}\partial^{\mu}$ ... expresión \ref{ecuacion.KG.2}:
\begin{equation}\label{ecuacion.KG.2}
\left(\Box + m^2\right)\phi=0.
\end{equation}
\end{solution}
\noindent\rule{\textwidth}{2.8pt}
\end{document}
0% Loading or .
You are about to add 0 people to the discussion. Proceed with caution.
Finish editing this message first!
Please register or to comment